You are on page 1of 18

Official Solutions F

MAA American Mathematics Competitions A


L
23rd Annual

AMC 10 B L
Tuesday, November 16, 2021

This official solutions booklet gives at least one solution for each problem on this year’s competition and shows
that all problems can be solved without the use of a calculator. When more than one solution is provided, this is
done to illustrate a significant contrast in methods. These solutions are by no means the only ones possible, nor
are they necessarily superior to others the reader may devise.
Teachers are encouraged to share copies of the problem booklet and official solutions with their students for edu-
cational purposes. All problems should be credited to the MAA AMC (for example, “2017 AMC 12 B, Problem
#21”). The publication, reproduction, or communication of the competition’s problems or solutions for revenue-
generating purposes requires written permission from the Mathematical Association of America (MAA).
Questions and comments about this competition should be sent to
amcinfo@maa.org
or
MAA American Mathematics Competitions
P.O. Box 471
Annapolis Junction, MD 20701.
The problems and solutions for this AMC 10 B were prepared by the
MAA AMC 10/12 Editorial Board under the direction of
Azar Khosravani and Carl Yerger, co-Editors-in-Chief.

© 2021 Mathematical Association of America


2 Fall 2021 AMC 10 B Solutions

1. What is the value of 1234 C 2341 C 3412 C 4123 ?

(A) 10;000 (B) 10;010 (C) 10;110 (D) 11;000 (E) 11;110

Answer (E): When the four numbers are added, the digits 1, 2, 3, and 4 appear exactly once in each column of
the addition problem, as shown in the figure.

1234
2341
3412
+ 4123

The digits sum to 10, producing a carry of 1 in each column. The sum of the four numbers is therefore 11;110.

2. What is the area of the shaded figure shown below?

0 1 2 3 4 5 6

(A) 4 (B) 6 (C) 8 (D) 10 (E) 12

Answer (B): This area can be computed as the area of the isosceles triangle with base 4 and height 5 minus
the area of the isosceles triangle with base 4 and height 2, namely
1 1
45  4  2 D 10 4 D 6:
2 2

OR

The region consists of two triangles with vertical base of length 3 and horizontal height 2. Therefore the area is
1
2  3  2 D 6:
2
2021 2020 p
3. The expression 2020 2021
is equal to the fraction q
in which p and q are positive integers whose greatest
common divisor is 1. What is p ?

(A) 1 (B) 9 (C) 2020 (D) 2021 (E) 4041

Answer (E): The given expression equals

2021 2021 2020 2020 20212 20202


  D
2020 2021 2021 2020 2020  2021
Fall 2021 AMC 10 B Solutions 3

.2021 C 2020/.2021 2020/


D
2020  2021
4041
D :
2020  2021

Because 4041 2  2020 D 1, it follows that 4041 and 2020 cannot have a common divisor greater than 1.
Similarly, because 4041 2  2021 D 1, it follows that 4041 and 2021 cannot have a common divisor greater
4041
than 1. Hence 20202021 is in simplest terms, and the requested numerator is 4041.

OR

Let n D 2020. Then the given fraction equals

nC1 n .n C 1/2 n2
D
n nC1 n.n C 1/ n.n C 1/
n2 C 2n C 1 n2
D
n.n C 1/ n.n C 1/
2n C 1
D :
n.n C 1/

Note that if d is a divisor of both a and a C b, then d is also a divisor of their difference, b. Because n and
n C 1 have no common divisors greater than 1, it follows that 2n C 1 can have no common divisors greater than
1 with either n or n C 1. Thus p D 2n C 1 D 4041.

4. At noon on a certain day, Minneapolis is N degrees warmer than St. Louis. At 4:00 the temperature in Min-
neapolis has fallen by 5 degrees while the temperature in St. Louis has risen by 3 degrees, at which time the
temperatures in the two cities differ by 2 degrees. What is the product of all possible values of N ?

(A) 10 (B) 30 (C) 60 (D) 100 (E) 120

Answer (C): At 4:00 the amount by which the temperature in Minneapolis exceeds the temperature in St. Louis
has decreased by 5 C 3 D 8. Therefore jN 8j D 2. The two solutions to this equation are N D 6 and N D 10.
The requested product is 6  10 D 60.

OR

Let M and S be the temperatures in the two cities at noon. It is given that M S D N . At 4:00 the temperatures
are M 5 and S C 3, respectively. The difference in temperature at 4:00 is therefore

j.M 5/ .S C 3/j D j.M S/ 8j D jN 8j:

The equation jN 8j D 2 has two solutions, N D 6 and N D 10. The requested product is 6  10 D 60.

n
5. Let n D 82022 . Which of the following is equal to 4
?

(A) 41010 (B) 22022 (C) 82018 (D) 43031 (E) 43032

Answer (E): Note that 8 D 23 , so


2022 3033
n D 82022 D 23 D 26066 D 22 D 43033 :

Thus
n 43033
D D 43032 :
4 4
4 Fall 2021 AMC 10 B Solutions

All the other choices are less than 43032 . Choices (A) and (D) have the same base but a lesser exponent. Choice
(B) is incorrect because 22022 < 42022 < 43032 . Choice (C) is incorrect because
 3 2018
82018 D 4 2 D 43027 < 43032 :

6. The least positive integer with exactly 2021 distinct positive divisors can be written in the form m  6k , where m
and k are integers and 6 is not a divisor of m. What is m C k ?

(A) 47 (B) 58 (C) 59 (D) 88 (E) 90

Answer (B): The number of positive integer divisors of the positive integer whose prime factorization is
p1e1 p2e2    pnen equals .e1 C 1/.e2 C 1/    .en C 1/. Because

2021 D 2025 4 D 452 22 D .45 C 2/.45 2/ D 47  43;

a number having 2021 divisors must be of the form p 2020 or p 46  q 42 , where p and q are distinct primes. This
is minimized by taking p D 2 in the first case, and p D 2 and q D 3 in the second case. Because

246  342 < 246  442 D 246C242 D 2130 < 22020 ;

the least such positive integer is

246  342 D 24  242  342 D 24  642 D 16  642 :

Therefore m C k D 16 C 42 D 58.

7. Call a fraction ab , not necessarily in simplest form, special if a and b are positive integers whose sum is 15. How
many distinct integers can be written as the sum of two, not necessarily different, special fractions?

(A) 9 (B) 10 (C) 11 (D) 12 (E) 13

Answer (C): The 14 special fractions are

1 2 3 4 5 6 7 8 9 10 11 12 13 14
; ; ; ; ; ; ; ; ; ; ; ; ; ;
14 13 12 11 10 9 8 7 6 5 4 3 2 1
which simplify to

1 2 1 4 1 2 7 8 3 11 13
; ; ; ; ; ; ; ; ; 2; ; 4; ; 14:
14 13 4 11 2 3 8 7 2 4 2
The fractions can be grouped as follows:
 The integers 2, 4, and 14 can be doubled to produce 4, 8, and 28, or paired to produce sums of 6, 16, and
18.
 The fractions with denominators of 2 are 21 , 23 , and 13
2
. They can be doubled to produce 1, 3, and 13, or
paired to produce sums of 2, 7, and 8.
1
 The fractions with denominators of 4 are 4
and 11
4
. Their sum is 3.
1 2
 The remaining fractions— 14 , , 4,
13 11
2 7
, , and 78 —cannot be added
3 8
to any fraction in the list to give an
integer.

This analysis produced 13 integer sums, but 3 and 8 appeared twice. Excluding the duplicates leaves 11 distinct
integers that can be written as the sum of two special fractions: 1, 2, 3, 4, 6, 7, 8, 13, 16, 18, and 28.
Fall 2021 AMC 10 B Solutions 5

8. The greatest prime number that is a divisor of 16;384 is 2 because 16;384 D 214 . What is the sum of the digits
of the greatest prime number that is a divisor of 16;383 ?

(A) 3 (B) 7 (C) 10 (D) 16 (E) 22

Answer (C): Observe that 16;383 D 214 1 D .27 C 1/.27 1/ D 129  127 D 3  43  127. Because all three
of these factors are prime, the greatest prime number that is a divisor of 16;383 is 127, and the requested sum of
digits is 1 C 2 C 7 D 10.

9. The knights in a certain kingdom come in two colors: 72 of them are red, and the rest are blue. Furthermore, 16
of the knights are magical, and the fraction of red knights who are magical is 2 times the fraction of blue knights
who are magical. What fraction of red knights are magical?
2 3 7 2 1
(A) (B) (C) (D) (E)
9 13 27 7 3

Answer (C): Let N be the number of knights in the kingdom. There are 27 N red knights and 57 N blue knights.
Let f represent the fraction of red knights who are magical. Then the fraction of blue knights who are magical
is 12 f , and the total number of magical knights is 61 N , so
2 1 5 1
f  N C f  N D N:
7 2 7 6
Multiplying both sides of the equation by 42 and dividing by N gives

12f C 15f D 7;
7
so f D 27
.

10. Forty slips of paper numbered 1 to 40 are placed in a hat. Alice and Bob each draw one number from the hat
without replacement, keeping their numbers hidden from each other. Alice says, “I can’t tell who has the larger
number.” Then Bob says, “I know who has the larger number.” Alice says, “You do? Is your number prime?”
Bob replies, “Yes.” Alice says, “In that case, if I multiply your number by 100 and add my number, the result is
a perfect square.” What is the sum of the two numbers drawn from the hat?

(A) 27 (B) 37 (C) 47 (D) 57 (E) 67

Answer (A): Based on Alice’s first statement, Bob can deduce that her number is not 1 or 40. Bob says that he
knows who has the larger number, which implies that his number must be 1, 2, 39, or 40. The number 2 is the
only prime among them, which tells Alice that Bob’s number is 2. Alice then says that her number added to 200
is a perfect square. The only perfect square between 201 and 240 is 152 D 225, so Alice’s number is 25 and the
sum of the two numbers is 25 C 2 D 27.

11. A regular hexagon of side length 1 is inscribed in a circle. Each minor arc of the circle determined by a side of
the hexagon is reflected over that side. What is the area of the region bounded by these 6 reflected arcs?
p p
5 3 p p 3 3
(A)  (B) 3 3  (C) 4 3 (D) 
2 2 2
p
C 3
(E)
2
Answer (B): Note that the reflected arcs do not overlap except at their endpoints. The area of the region can be
found by subtracting from the area of the hexagon the difference between the areas of the circle and the hexagon.
This is equivalent to twice the area of the hexagon minus the area of the circle. Therefore the requested area is
p
12  3 p
26   12 D 3 3 :
4
6 Fall 2021 AMC 10 B Solutions

12. Which of the following conditions is sufficient to guarantee that integers x, y, and z satisfy the equation

x.x y/ C y.y z/ C z.z x/ D 1 ‹

(A) x > y and y D z (B) x D y 1 and y D z 1


(C) x D z C 1 and y D x C 1 (D) x D z and y 1Dx
(E) x C y C z D 1

Answer (D): The given equation is equivalent to 2x.x y/ C 2y.y z/ C 2z.z x/ D 2 , which can be
rewritten as .x y/2 C .y z/2 C .z x/2 D 2. This equation has an integer solution if and only if two of
the squares are 1 and one is 0. This in turn means that two of the variables must be equal and the third must
differ from this common value by 1. Choice (D) gives one instance of this, and the other choices do not imply
this condition. Specifically, choice (A) fails when x D 2, y D 0, and z D 0 because the left-hand side of the
original equation equals 4; choice (B) fails when x D 1, y D 2, and z D 3 because the left-hand side equals 3;
choice (C) fails when x D 1, y D 2, and z D 0 because the left-hand side equals 3 ; and choice (E) fails when
x D 2, y D 0, and z D 1 because the left-hand side equals 7.

13. A square with side length 3 is inscribed in an isosceles triangle with one side of the square along the base of
the triangle. A square with side length 2 has two vertices on the other square and the other two on sides of the
triangle, as shown. What is the area of the triangle?

1 1 3 1 3
(A) 19 (B) 20 (C) 21 (D) 22 (E) 23
4 4 4 2 4
Fall 2021 AMC 10 B Solutions 7

Answer (B): Label the vertices as shown in the diagram.

C
B
E
D

F G

Then 4ABC , 4CDE, and 4EF G are similar. Because CD D 2 and DE D 3 2 2 D 12 , the lengths of the legs
of each of these triangles are in the ratio of 4 to 1. It follows that F G D 34 , so the base of the isosceles triangle
has length 34 C 3 C 43 D 92 . Similarly, because BC D 1, similar triangles give AB D 4. It follows that the
altitude of the isosceles triangle is 4 C 2 C 3 D 9. The area of the triangle is then given by 21  29  9 D 20 14 .

OR

Place the triangle in a coordinate plane withthe base on the x-axis and the apex on the positive y-axis. The
upper right vertex of the large square is 23 ; 3 , and the upper right vertex of the small square is .1; 5/. The side
of the triangle in the first quadrant has slope
3 5
3
D 4;
2
1

and its equation is y D 9 4x. Thus the side intersects the x-axis at 94 ; 0 and the y-axis at .0; 9/. Therefore


the triangle has base 2  49 D 29 and altitude 9, so its area is 12  92  9 D 20 14 .

14. Una rolls 6 standard 6-sided dice simultaneously and calculates the product of the 6 numbers obtained. What is
the probability that the product is divisible by 4 ?
3 57 59 187 63
(A) (B) (C) (D) (E)
4 64 64 192 64
Answer (C): The product will not be divisible by 4 precisely when all 6 rolls are odd, or exactly one of them
is equal to either 2 or 6 and the rest are odd. The probability of this complementary event is
 6    5
1 1 1 5
C6  D :
2 3 2 64
5 59
The requested probability is therefore 1 64
D 64
.

15. In square ABCD, points P and Q lie on AD and AB, respectively. Segments BP and CQ intersect at right
angles at R, with BR D 6 and PR D 7. What is the area of the square?
8 Fall 2021 AMC 10 B Solutions

D C

R
6

A Q B

(A) 85 (B) 93 (C) 100 (D) 117 (E) 125

Answer (D): Because ∠RBC is complementary to both ∠RCB and ∠PBA, those two angles are congruent.
Therefore 4BAP Š 4CBQ by ASA, so CQ D BP D 13. Let d D CR; then QR D 13 d , so the Altitude-
to-Hypotenuse Theorem yields 62 D d.13 d /, which has solutions d D 4 and d D 9. Because QR < RC ,
in fact d D 9. It follows that the area of the square is

BC 2 D BR2 C RC 2 D 62 C 92 D 117:

OR

Let c D BQ and s D AB. Because 4BRQ is similar to 4BAP , it follows that 6c D 13 s


, so cs D 78. As
2 2 2
before, CQ DpBP D 13, so by the Pythagorean Theorem, s C c D 169.
p Then .s C c/ D 169 C 2  78 D 325,
so s C c D 5p 13. Similarly .s c/2 D 169 2  78 D 13, so s c D 13. Solving this system of equations
yields s D 3 13, and the area of the square is s 2 D 117.

16. Five balls are arranged around a circle. Chris chooses two adjacent balls at random and interchanges them. Then
Silva does the same, with her choice of adjacent balls to interchange being independent of Chris’s. What is the
expected number of balls that occupy their original positions after these two successive transpositions?

(A) 1:6 (B) 1:8 (C) 2:0 (D) 2:2 (E) 2:4

Answer (D): Label the balls in order 1 through 5. Assume without loss of generality that the first transposition
is 1 $ 2, resulting in the order 21345. The following table shows the results of the 5 equally likely second
transpositions.

2nd transposition result balls in original position count


2$1 12345 1,2,3,4,5 5
1$3 23145 4,5 2
3$4 21435 5 1
4$5 21354 3 1
5$2 51342 3,4 2

The expected number of balls that occupy their original positions is the average of the numbers in the last
column, namely 15 .5 C 2 C 1 C 1 C 2/ D 2:2.
Fall 2021 AMC 10 B Solutions 9

OR

Let Xi be the random variable that has the value 1 if ball i occupies its original position after the two successive
transpositions and the value 0 if it does not. The problem asks for the expected value of the sum X1 C X2 C
X3 CX4 CX5 , which, by linearity of expectation, is the sum of the expected values. The probability that Xi D 1
equals 25  15 C 35  35 D 1125
, because this will happen if ball i is involved in the first transposition and the second
transposition involves the same two balls, or ball i is involved in neither transposition. Therefore the expected
value of each Xi is 11 25
, and the expected value of their sum is 5  1125
D 11
5
D 2:2.

17. Distinct lines ` and m lie in the xy-plane. They intersect at the origin. Point P . 1; 4/ is reflected about line `
to point P 0 , and then P 0 is reflected about line m to point P 00 . The equation of line ` is 5x y D 0, and the
coordinates of P 00 are .4; 1/. What is the equation of line m ?

(A) 5x C 2y D 0 (B) 3x C 2y D 0 (C) x 3y D 0


(D) 2x 3y D 0 (E) 5x 3y D 0

Answer (D): The reflection through ` followed by the reflection through m is equivalent to a rotation about
the intersection of the two lines by twice the angle formed by the two lines. As the result of the two reflections
in the present case, P was rotated by 90ı clockwise around the origin to P 00 .4; 1/. Therefore the line ` must be
rotated by 45ı clockwise about the origin to obtain line m.
The slope of line ` is 5, and a quick sketch shows that the slope of line m is a positive number less than 5. The
equation of m can be calculated if given one point on m other than the origin O. Point A.1; 5/ is on line `.
Let B.x; y/ be the intersection of line m with the line through A perpendicular to line `. Then 4BAO is an
isosceles right triangle with right angle at A. Then the slope of line AB is 51 , so y 5 D 51 .x 1/. Point B
is on the circle with radius OA centered at A. Therefore .x 1/2 C .y 5/2 D 26. The two solutions of this
pair of equations are .6; 4/ and . 4; 6/. Because B is in the first quadrant, it must be .6; 4/, so the slope of m
is 23 , and its equation can be written 2x 3y D 0.

l : y = 5x

(x – 1)2 + ( y – 5)2 = 26

A(1, 5)
P(– 1, 4)
P¢ B(6, 4)

P¢¢(4, 1)
O (0, 0)
2
m :y = 3x
10 Fall 2021 AMC 10 B Solutions

18. Three identical square sheets of paper each with side length 6 are stacked on top of each other. The middle sheet
is rotated clockwise 30ı about its center and the top sheet is rotated clockwise 60ı about its center, resulting
pin
the 24-sided polygon shown in the figure below. The area of this polygon can be expressed in the form a b c,
where a, b, and c are positive integers, and c is not divisible by the square of any prime. What is a C b C c ?

(A) 75 (B) 93 (C) 96 (D) 129 (E) 147

Answer (E): Let O be the center of the polygon, and label 11 points as shown in the figure. Let a D AB D
BC .

C E
D
A B F G
J K

I H

ı
p p
Triangle BCK p is a 30 – 60 – 90 triangle, so BK D 2a and CK D KG D a 3. Then AG D 3a C a 3 D 6,
so a D 3 3. The area of the 24-sided polygon can be computed as 12 times p the area of kite OBCD. The
longer diagonal of this kite is OC , half of a diagonal of the square, so OC D 3p 2. The shorter diagonal of the
kite is BD, the hypotenuse of isosceles right triangle BCD with leg a D 3 3. The area of a kite is half the
product of the lengths of its diagonals, so the area of the 24-sided polygon is
1 p  p p p
12  3 2 3 3 2 D 108 36 3:
2
Therefore a C b C c D 108 C 36 C 3 D 147.

OR
p
Label the points as in the first solution, where it was shown that a D AB D BC D 3 3. The area of
the 24-sided polygon can be found by adding to the area of square AGHI the areas of 8 triangles congruent to
Fall 2021 AMC 10 B Solutions 11

4BCK and then subtracting the areas of 4 triangles congruent to 4DJK. The area of the square is 36. The
area of 4BCK is
p
 p 2 p
2
a 3 3 3 3 p
D D 6 3 9:
2 2
To findpthe area of 4DJK,
p note that it is an isosceles triangle with vertex angle 120ı and with base JK D
6 2a 3 D 12 6 3. The length of the altitude to the base is then
p
12 6 3 p
p D 2 3 3:
2 3
Thus the area of 4DJK is
1  p   p  p
 12 6 3  2 3 3 D 21 3 36:
2
Finally, the area of the polygon is
 p   p  p
36 C 8 6 3 9 4 21 3 36 D 108 36 3:

Therefore a C b C c D 147, as above.

19. Let N be the positive integer 7777 : : : 777, a 313-digit number where each digit is a 7. Let f .r/ be the leading
digit of the rth root of N . What is f .2/ C f .3/ C f .4/ C f .5/ C f .6/ ?

(A) 8 (B) 9 (C) 11 (D) 22 (E) 29

Answer (A): Because 10r is written as a 1 followed by r zeros, the rth root of any number smaller than 10r
when written as a decimal has only one digit to the left of the decimal point. Extending this reasoning, the
leading digit of the rth root of N is the same as the leading digit of the rth root of the integer n that has as digits
a number of 7s equal to the remainder when 313 is divided by r. For example, the fifth root of N has the same
leading digit as the fifth root of 777, because the remainder of 313 when divided by 5 is 3. Because 35 D 243
and 45 D 1024, the leading digit is 3.
Using this methodology, it follows that f .2/ D 2, f .3/ D 1, f .4/ D 1, f .5/ D 3, and f .6/ D 1. The
requested sum is 2 C 1 C 1 C 3 C 1 D 8.

20. In a particular game, each of 4 players rolls a standard 6-sided die. The winner is the player who rolls the highest
number. If there is a tie for the highest roll, those involved in the tie will roll again and this process will continue
until one player wins. Hugo is one of the players in this game. What is the probability that Hugo’s first roll was
a 5, given that he won the game?
61 367 41 185 11
(A) (B) (C) (D) (E)
216 1296 144 648 36
Answer (C): First observe that if q players tie on the initial roll, the probability that any one of these q players
will ultimately win is q1 . Let N be the value of Hugo’s first roll.
Consider four cases based on the number of highest scoring rolls in the first round. The probability that Hugo
will roll a number larger than the other three players is

.N 1/3 N3 3N 2 C 3N 1
D :
63 216
The probability that Hugo will tie one other player, beat the other two players, and ultimately win is

1 .N 1/2 1 N 2 2N C 1
3   D :
6 62 2 144
12 Fall 2021 AMC 10 B Solutions

Similarly, the probability that Hugo will tie two other players, beat the other player, and ultimately win is
1 N 1 1 N 1
3   D :
62 6 3 216
Finally, the probability that Hugo will tie all three players and ultimately win is
1 1 1
 D :
63 4 864

The sum of these four probabilities is


4N 3 6N 2 C 4N 1
:
864
1 15 65 175 369 671
Evaluating this expression for N from 1 to 6 yields 864 , 864 , 864 , 864 , 864
, and 864
, respectively. Hence the
probability that Hugo rolled a 5 on his initial roll given that he won is
369 369 41
D D :
1 C 15 C 65 C 175 C 369 C 671 1296 144

OR

This can also be solved using Bayes’ Theorem. The probability that Hugo rolled a 5 on his initial roll given that
he won, written P .5 j W/, is
41 1
P .W j 5/  P .5/  41
D 96 1 6 D ;
P .W/ 4
144
41 369
where 96
is the number 864
computed in the solution above for N D 5.

OR

This solution is an application of the following general formula. Suppose there were k players and an n-sided
die was rolled. Then, for any 1  m  n, using the notation in the second solution,

mk .m 1/k
P .Hugo’s first roll was m j Hugo won/ D :
nk
54 44 41
In the context of the original problem, this makes the answer 64
D 144
.
Two proofs are presented. The first proof is straightforward but somewhat computational. The second proof is
harder to motivate but more elegant.

Proof 1: Algebra For each 0  j  k 1, let Aj be the event that j of the remaining k 1 players rolled an
m while the other k 1 j players rolled less than m. Note that
  j 
m 1 k 1 j
 
k 1 1
P .Aj / D :
j n n
After this first round, the remaining j C 1 players went into the tiebreaker, and the probability of winning there
1
is j C1 by symmetry. As a result,

k
X1 1
P .Hugo won j Hugo’s first roll was an m/ D P .Aj / 
j C1
j D0
k
X1   j 
m 1 k 1 j
 
1 k 1 1
D
j C1 j n n
j D0
Fall 2021 AMC 10 B Solutions 13

k
X1   j 
m 1 k 1 j
 
1 k 1
D
k j C1 n n
j D0
k    j 
m 1 k j

nX k 1
D
k j n n
j D1
 k  k !
n 1 m 1 m 1
D C
k n n n
mk .m 1/k
D ;
k nk 1

where the second to last step uses the Binomial Theorem. An application of Bayes’ Theorem as in the second
solution yields the desired result:
 k 
m .m 1/k
k nk 1
 n1
P .Hugo won j Hugo’s first roll was an m/ D 1
k
mk .m 1/k
D :
nk

Proof 2: Combinatorics Observe that, given that Hugo won, Hugo’s roll was at most m if and only if ev-
eryone’s rolls were at most m. Indeed, Hugo’s roll must be the highest roll to even have a chance at winning.
Therefore

P .Hugo’s first roll was  m j Hugo won/ D P .all first rolls were  m j Hugo won/
 m k
D P .all first rolls were  m/ D ;
n
where the fact is used that the events “all first rolls were  m” and “Hugo won” are independent. Therefore the
probability that Hugo’s first roll was an m given that Hugo won is

P .Hugo’s first roll was  m j Hugo won/ P .Hugo’s first roll was  m 1 j Hugo won/;

which equals
mk .m 1/k
;
nk
as desired.

21. Regular polygons with 5, 6, 7, and 8 sides are inscribed in the same circle. No two of the polygons share a
vertex, and no three of their sides intersect at a common point. At how many points inside the circle do two of
their sides intersect?

(A) 52 (B) 56 (C) 60 (D) 64 (E) 68

Answer (E): Consider a regular m-gon and a regular n-gon, with m  n, inscribed in the same circle with no
>
shared vertices. If A and B are adjacent vertices of the m-gon, then minor arc AB contains at least one vertex
of the n-gon. Thus side AB intersects exactly two sides of the n-gon inside the circle, and the two polygons
intersect in exactly 2m points. It follows that the pentagon intersects the hexagon, the heptagon (the regular
polygon with 7 sides), and the octagon in 10 points each; the hexagon intersects the heptagon and the octagon in
12 points each; and the heptagon intersects the octagon in 14 points. The total number of points of intersection
is 3  10 C 2  12 C 14 D 68.
14 Fall 2021 AMC 10 B Solutions

A B

m-gon

22. For each integer n  2, let Sn be the sum of all products j k, where j and k are integers and 1  j < k  n.
What is the sum of the 10 least values of n such that Sn is divisible by 3 ?

(A) 196 (B) 197 (C) 198 (D) 199 (E) 200

Answer (B): For n  2, let

.n 1/n2
Un D .1 C 2 C 3 C    C .n 1//  n D :
2
Then Sn D Sn 1 C Un for n  3. Note that Un is divisible by 3 if n  0 or 1 .mod 3/; and if n  2 .mod 3/,
then .n 1/n2  1 .mod 3/ and is even, so Un  2 .mod 3/. Hence SnC3  Sn C 2 .mod 3/ for n  2. It
is readily verified that S2  S3  S4  2 .mod 3/, so S5  S6  S7  1 .mod 3/ and S8  S9  S10  0
.mod 3/, and it follows that Sn is divisible by 3 if and only if n  0 or ˙1 .mod 9/. Thus the sum of the 10
least values of n that satisfy the required condition is

8 C 9 C 10 C 17 C 18 C 19 C 26 C 27 C 28 C 35 D 197:

OR

The sum of the products j k as j and k run independently from 1 to n is


 2
2 n.n C 1/
.1 C 2 C    C n/ D :
2

To eliminate the cases in which j D k, subtract

n.n C 1/.2n C 1/
12 C 22 C    C n2 D :
6
Thus
X n2 .n C 1/2 n.n C 1/.2n C 1/ .n 1/n.n C 1/.3n C 2/
jk D D :
4 6 12
1j n
1kn
j ¤k

For a given pair j; k with j ¤ k either j < k or j > k, but their product is the same in either order. To impose
the condition j < k, it suffices to divide by 2. Thus

.n 1/n.n C 1/.3n C 2/
Sn D :
24
There is one factor of 3 in the denominator. For any n, exactly one of n C 1, n, n 1 is divisible by 3, and
3n C 2 is not divisible by 3. In order that Sn be divisible by 3 it is necessary and sufficient that the factor that is
divisible by 3 should in fact be divisible by 9. That is, n  0 or ˙ 1 .mod 9/, and the answer can be calculated
as above.
Fall 2021 AMC 10 B Solutions 15

23. Each of the 5 sides and the 5 diagonals of a regular pentagon are randomly and independently colored red or
blue with equal probability. What is the probability that there will be a triangle whose vertices are among the
vertices of the pentagon such that all of its sides have the same color?
2 105 125 253
(A) (B) (C) (D) (E) 1
3 128 128 256
Answer (D): It will be easier to compute the probability that no monochromatic triangles exist. Suppose one
of the vertices, say A, has 3 segments of the same color connecting it to 3 other vertices, say B, C , and D.
If one of the edges of 4BCD has the same color as edges AB, AC , and AD, then a monochromatic triangle
exists. Otherwise, 4BCD forms a monochromatic triangle of the other color. Therefore in order for there to be
no monochromatic triangles, each vertex must be incident to exactly 2 red and 2 blue segments. This is possible
only if the coloring creates a loop of 5 segments all colored red and a loop of 5 segments all colored blue.

There are 4Š
2
D 12 choices for the red loop because the loop can always be viewed as starting at a particular
vertex and can go in two different directions. There are 210 different colorings of the 10 segments. Therefore
the requested probability is
12 3 253
1 10
D1 D :
2 256 256

Note: The corresponding probability if one uses a hexagon instead of a pentagon is 1. More generally, Ramsey’s
Theorem states that for every integer k  3 and every integer c  2, there is an integer n such that if all the sides
and diagonals of the regular n-gon are colored using c colors, then there will be a subset of k of the vertices
such that all the segments joining points in that subset have the same color.

24. A cube is constructed from 4 white unit cubes and 4 blue unit cubes. How many different ways are there to
construct the 2  2  2 cube using these smaller cubes? (Two constructions are considered the same if one can
be rotated to match the other.)

(A) 7 (B) 8 (C) 9 (D) 10 (E) 11

Answer (A): Rotate the cube so that the number of small blue cubes showing in the front face is maximized.
 If the front face contains four blue cubes, this gives 1 possible construction.
 If the front face contains three blue cubes, then the back face must contain one blue cube, which can be
in any of 4 positions. None of these constructions can be rotated into any other, so this gives 4 possible
constructions. Here is why the four placements of the one blue cube on the back face lead to different
patterns. Assume without loss of generality that the sole white cube on the front face is in the upper right.
There is a second face with three blue cubes if and only if the back cube is placed in any position on the
back face except for the upper right. Hence if the blue cube on the back face is placed at the upper right,
16 Fall 2021 AMC 10 B Solutions

then there will be a blue cube adjacent to no other blue cubes, and this does not occur with any other
placement of the blue cube on the back face. If the blue cube on the back face is placed at the lower left,
then there is one blue cube with three blue neighbors, but not if it is placed anywhere else. This leaves
two cases, namely when the blue cube on the back face is placed at the upper left or lower right. In either
of these cases, if the cube is rotated so that the unique other face having three blue cubes is placed in the
front with the sole white cube on that face in the upper right, then the same configuration of blue and white
cubes is obtained, rather than the other one. This gives 4 possible constructions.
 Otherwise, each face of the large cube contains exactly two blue cubes. If some face contains two blue
cubes in adjacent positions in a row or column, then rotate the cube to make that face the front face. Then
the back face must contain two blue cubes in the diagonally opposite row or column, respectively. This
gives 1 possible construction.
 Otherwise, no blue cubes are in adjacent positions, and every face of the large cube is colored blue and
white in checkerboard fashion. There is just 1 way to do this.
Thus the total number of possible constructions is 1 C 4 C 1 C 1 D 7. The figure below shows the four possible
cases.

25. A rectangle with side lengths 1 and 3, a square with side length 1, and a rectangle R are inscribed inside a larger
square as shown. The sum of all possible values for the area of R can be written in the form m n
, where m and n
are relatively prime positive integers. What is m C n ?

1
1
1
1 R

1
3

3
1

(A) 14 (B) 23 (C) 46 (D) 59 (E) 67

Answer (E): Label the diagram as shown below, where O, P , and Q are the feet of the perpendicular segments
to the respective sides. Let x D AE and y D AF .
Fall 2021 AMC 10 B Solutions 17

A y F P L B
x
E 1
1
1
1 G R M
O
H
1 N
3 Q
I
K
3
1

D J C

Because
4FAE Š 4EOH Š 4IOH Š 4KCJ Š 4FP G;
it follows that EO D IO D KC D y and CJ D AE D x. Furthermore, 4JDI is similar to all these triangles
with scale factor 3, so ID D 3x and DJ D 3y. Therefore

3y C x D DC D DA D 2y C 4x;

so y D 3x. Applying the Pythagorean Theorem to 4AEF yields x D p1 .


10
Now set PL D ax and LB D bx for some positive real numbers a and b. Note that MQ D P G D y D 3x,
and because 4GPL  4LBM , it follows that BM D 13 abx. Furthermore, 4NQK  4KCJ , and because
NQ D PL D ax, it follows that QK D 13 ax. Therefore

1 1
3x C ax C 3x C abx D BC D AD D 10x;
3 3
which simplifies to a.b C 1/ D 12. Because PB D AB AP D 6x, it follows that a C b D 6. Solving these
equations simultaneously for a and b shows that .a; b/ equals either .3; 3/ or .4; 2/.
Finally, observe that the area of R is
s
 2
p 1
LG  LM D .ax/2 C .3x/2  .bx/2 C abx
3
s  
1 2 b
D .a2 C 9/ 1 C a  :
9 10

When .a; b/ D .3; 3/, this area equals 59 ; and when .a; b/ D .4; 2/, this area equals 53 . The sum of these two
areas is 59 C 53 D 15
52
, and the requested sum of numerator and denominator is 52 C 15 D 67.

Problems and solutions were contributed by Theodore Alper, Sophie Alpert, David Altizio, Risto Atanasov, Bela
Bajnok, Chris Bolognese, Miklos Bona, Silva Chang, Barbara Currier, Steven Davis, Steve Dunbar, Zuming Feng,
Zachary Franco, Peter Gilchrist, Andrea Grof, Jerrold Grossman, Thomas Howell, Chris Jeuell, Daniel Jordan,
Jonathan Kane, Ed Keppelman, Azar Khosravani, Sergey Levin, Joseph Li, Ioana Mihaila, Hugh Montgomery, Lucian
Sega, Matyas Sustik, Zsuzsanna Szaniszlo, Agnes Tuska, David Wells, Kathleen Wong, Carl Yerger, and Jesse Zhang.
MAA Partner Organizations
We acknowledge the generosity of the following
organizations in supporting the MAA AMC and
Invitational Competitions:

Akamai Foundation

Army Educational Outreach Program

Art of Problem Solving

AwesomeMathGirls.org

Casualty Actuarial Society

Jane Street Capital

MathWorks

Society for Industrial and Applied Mathematics

TBL Foundation

The D. E. Shaw Group

Tudor Investment Corporation

Two Sigma

You might also like